LSAT and Law School Admissions Forum

Get expert LSAT preparation and law school admissions advice from PowerScore Test Preparation.

 Administrator
PowerScore Staff
  • PowerScore Staff
  • Posts: 8949
  • Joined: Feb 02, 2011
|
#23491
Complete Question Explanation

Justify the Conclusion-CE. The correct answer choice is (B)

The stimulus tells us that large finches are able to better survive during drought periods because of their ability to crack open large, hard seeds. According to the stimulus, the large finches do worse in wet periods because additional growth produces more small seeds, which have to be consumed in enormous numbers to meet the large finches' energy needs. While the stimulus addresses the increase in small seeds during wet periods, it fails to address whether there is a concurrent decrease in the production of large seeds. Such a decrease would be sufficient to justify conclusion.

Answer Choice (A): This answer choice, if anything, weakens the stimulus argument by disputing the information provided indicating that drought years are more favorable to large finches because they can crack open and store the larger seeds.

Answer Choice (B): This is the correct answer choice. We need to know more than just the fact that more small seeds are produced during dry periods. We need to also know that less large, hard seeds are being produced if we are going to get to the stated conclusion.

Answer Choice (C): This does nothing to support the stimulus argument. The hole in the argument is that fact that we do not know whether the wet periods result in a decrease in large seeds along with the increase in small seeds. This answer does not fill that hole.

Answer Choice (D): This answer choice may provide support for the overall accuracy of the date produced in the study. However, it does nothing to fill the hole in the argument regarding the study, and is therefore insufficient to justify the conclusion.

Answer Choice (E): While this information supports the conclusion that small birds do better when there are small seeds, it does nothing to fill the hole in the argument and is insufficient to justify the conclusion.
 jrc3813
  • Posts: 53
  • Joined: Apr 16, 2017
|
#35905
Isn't this question really an assumption question and not a justify the conclusion question? The stem asks what "must be assumed" in order to justify the conclusion. Moreover, the answer choice B does not 100% prove the argument, it's only necessary. The conclusion is that climatic variations cause a major difference in survival rates of small AND large birds. Answer choice B is necessary to show that some large birds can't survive in rainy years, but it doesn't tell us anything about small birds. Can they eat small seeds? In droughts we don't even know that small birds can't survive. All we know is that they can't eat the large seeds. A justify answer would seem to be more along the lines of "large finches can only eat large seeds and small birds can only eat small seeds. When there's drought there's significantly fewer small seeds relative to large seeds and the opposite during rainy years."

This question really confused me because of the language in the stem. But when I understood it as an assumption question it made a lot more sense.
 Jon Denning
PowerScore Staff
  • PowerScore Staff
  • Posts: 907
  • Joined: Apr 11, 2011
|
#36115
Hi jrc,

Thanks for the question—it's a good (and interesting) one! I want to make a few different points about this question and the issue you raise, as there's more to unpack here than meets the eye :0

First, I should note that this question appeared on the very first LSAT of the "modern era," in June 1991. That's PrepTest 1, so named because it represented a significant overhaul of the test from the version that appeared from June 1989-Feb 1991 (which was itself something of a transitional exam from an even more distinct form used since June 1982), with four scored sections instead of just three, and a new scoring scale of 120-180 instead of the decade-old 10-48 range.

I say that to emphasize an important detail: the test items in the very early 90s can occasionally, but rightly, be described as...imperfect. Not invalid mind you, but not held to quite the same standard of clarity and precision that questions from more recent exams exhibit. Like a dress shirt with the price marked down because one sleeve is a quarter-inch shorter than the other: entirely wearable, and probably unnoticeable unless you're closely measuring. But still not quite right.

Here's what I mean when it comes to #22.

I'm of the opinion that the test makers fully meant for this to be an Assumption question (as I'll discuss shortly), but used language that muddied that intent and invited understandable confusion, even fair debate, over the Assumption/Justify issue.

Why do I think that? Well, a big part of it is that this is the only instance since this test where the word "justify" has been used in the stem of a question that's arguably (or definitely) Assumption. Seriously. Hasn't happened since this question. They've phrased things similarly for other Assumption questions and used "enables the conclusion to be properly drawn," but not "justify." (That distinction is a little weighty for our purposes here; just suffice it to say that when something happened on the very first new exam but never again that means they found cause to avoid repeating it)

So that's my interpretation of where this categorization grey area comes from. It's a unique instance on a nascent exam and LSAC seems to have learned its lesson, improving the language used going forward.

Now, to the question itself.

I would argue—and we may get some healthy disagreement in subsequent replies, which is what makes this fun!—that this is in fact an Assumption question. Is it worded well for Assumption? Not particularly. Would you see this exact stem come up again and force you to make the Assumption or Justify distinction? Not if the last 26 years are any indication. But given the nature of both (soft) necessity in the question stem—"must be assumed in order to"—and a correct answer choice that is required for the conclusion but doesn't exactly prove it, to me we have to call this Assumption.

Specifically, answer choice (B), where rainy weather results in fewer large, hard seeds being produced, is a requirement of this argument (if rainy weather doesn't diminish the large seeds available then the larger finches wouldn't have to eat the smaller seeds and could be fine), but not proof that the argument is true (even if there are fewer, like maybe 1 less, large seeds in rainy weather, that wouldn't prove the large finches would have to eat small seeds to the point that they'll begin to die off). From this it seems clear that, question stem aside, LSAC wanted this to be an Assumption question. And test maker intent is ultimately the final word when it comes to how to view the LSAT.

Note: I hate (and strenuously avoid as I test) classifying questions based on the right answer choice. You should be able to label questions accurately based on the stimulus and question stem alone, using a prephrase from that analysis to then sort answers into right and wrong. Having to include the answers in order to determine type, or even know the right answer and its role before concluding what the question is, well that's not the way you want to tackle the LSAT. I'll grant that it's useful for us here in hindsight to figure out what the test makers intended, but as a test-taking strategy I'd advise against it

Again, I can't imagine you'd see this stem again so it's something of a moot point, but if you do find yourself on the fence between Assumption and Justify with other question stems the distinction can typically be made by this: Assumptions deal in what the argument and author require/need/must do or believe, while Justify is all about the conclusion being correct/properly drawn/valid. That tends to be sufficient to keep them recognizably separate, and would have me leaning towards Assumption for this example too.

Finally, I may go back in to the original explanation above (and classifications elsewhere) and recategorize this based on what I've just said. Regardless, hopefully that helps put your mind at ease a bit :)
 Mark83
  • Posts: 32
  • Joined: Sep 22, 2017
|
#40216
Hi, I was able to answer this question just by breaking it down in my head and eliminating answers. However, I would like to know how would one apply the mechanistic method (with corresponding diagram) to this type of justify question, as it differs in form from the kind of justify the conclusion questions discussed in the book in relation to the mechanistic method. Are there some questions that are better to use the formula on and some you should just do intuitively?
 Francis O'Rourke
PowerScore Staff
  • PowerScore Staff
  • Posts: 471
  • Joined: Mar 10, 2017
|
#40289
Hi Mark,

I would agree with Jon Denning above that this is closer to an assumption question, so that is exactly how I treated it.

After reading the stimulus, I noticed that there was a gap that needed to be filled in the argument. The speaker told us that during droughts, the large finches survive by eating large, hard seeds, but we have no idea of knowing what happens to these seeds during the rainy years. Why can't these finches continue to eat the large, hard seeds?

If you see that gap or oversight in the speaker's reasoning, you can make a good guess that the speaker must be assuming that the large, hard seeds are not as widely available during the rainy months. This is exactly what answer choice (B) tells us. We can confirm it by using the assumption negation technique. I would phrase the negation of answer choice (B) as follows:
A lengthy rainy period results in the same number of or more large, hard seeds.
This negated version of the answer choice definitively weakens the conclusion, so we have our correct answer.
 Mark83
  • Posts: 32
  • Joined: Sep 22, 2017
|
#40293
Francis O'Rourke wrote:Hi Mark,

I would agree with Jon Denning above that this is closer to an assumption question, so that is exactly how I treated it.

After reading the stimulus, I noticed that there was a gap that needed to be filled in the argument. The speaker told us that during droughts, the large finches survive by eating large, hard seeds, but we have no idea of knowing what happens to these seeds during the rainy years. Why can't these finches continue to eat the large, hard seeds?

If you see that gap or oversight in the speaker's reasoning, you can make a good guess that the speaker must be assuming that the large, hard seeds are not as widely available during the rainy months. This is exactly what answer choice (B) tells us. We can confirm it by using the assumption negation technique. I would phrase the negation of answer choice (B) as follows:
A lengthy rainy period results in the same number of or more large, hard seeds.
This negated version of the answer choice definitively weakens the conclusion, so we have our correct answer.
Thanks Francis. That's pretty much what I did was fill in the missing assumption, because I couldn't think of a quick or obvious way to break it down mechanistically.
 T.B.Justin
  • Posts: 194
  • Joined: Jun 01, 2018
|
#60179
Administrator wrote:Complete Question Explanation

Justify the Conclusion-CE. The correct answer choice is (B)


Answer Choice (B): This is the correct answer choice. We need to know more than just the fact that more small seeds are produced during dry periods. We need to also know that less large, hard seeds are being produced if we are going to get to the stated conclusion.
\
I believe this is supposed to be, "We need to know more than just the fact that more small seeds are produced during rainy periods.

The way it was original worded makes me think that this answer choice means since there are less large, hard seeds there must be more small seeds produced during dry periods.
 whomer
  • Posts: 3
  • Joined: Jan 29, 2019
|
#62261
I have two questions regarding the first problem in the Justify the Conclusion chapter in the LRQTT. Here's the problem:

A fourteen-year study of finches... [remainder of problem removed due to LSAC copyrights]

1. How do you solve this problem using conditional reasoning in the same way that the LRB does in the justify chapter?
Here's what I tried, which I'm confident is incorrect:

Premise 1: "During droughts, more members of large finch species survive because their bills are large enough to crack large, hard seeds, giving them a food supply unavailable to smaller birds."
Conditional 1: If there is a drought, then more large finch species survive. So D :arrow: LF

Premise 2: In rainy years, fewer members of the large finch species survive because the additional moisture fosters the growth of plants that produce small seeds. The larger finch varieties have to consume enormous numbers of small seeds to meet their energy demands, and some just cannot eat them fast enough.
Conditional 2: If there is no drought, fewer large finch species survive. So D Negated :arrow: LF negated

Conclusion: ...there is a definite relationship between climate and the population size of finch species that thrive at various times.
I figure "change in weather" can be designated as the lack or presence of a drought. If there is a drought than more large finch species survive (so a repeat of conditional 1) and if there isn't a drought then fewer large finch species survive (a repeat of conditional 2).

These relationships don't seem to allow for the conditional chains the LRB created for the justify questions at the end of chapter 10.

2. Does my inability to map these relationships indicate a weak understanding of some of the basics of conditional reasoning, or is it just a particularly difficult question to map? I've returned to the conditional reasoning chapter in the LRB a few times, but maybe it's time to go back again.

Thanks for the help!
User avatar
 Dave Killoran
PowerScore Staff
  • PowerScore Staff
  • Posts: 5973
  • Joined: Mar 25, 2011
|
#62264
Hi Whomer,

Thanks for the question! This problem is discussed over at lsat/viewtopic.php?t=9234.

We have separate discussion areas for every single released LSAT question. If you do a quick search for the first few words, you'll find them almost immediately. That might save you some time if we've explained the problem already.

Shortly we'll move your post over to that thread...

Thanks!
 whomer
  • Posts: 3
  • Joined: Jan 29, 2019
|
#62268
Thank you Dave! Apologies for posting in the wrong place.

I had seen that discussion, but it doesn't really map out the conditional relationships in the stem. Instead, it mostly concludes that the question should be treated as more of an assumption than as a justify.

So to my original question, do you think there's a way to map out those relationships, and if so, what do they look like?

Get the most out of your LSAT Prep Plus subscription.

Analyze and track your performance with our Testing and Analytics Package.